summaryrefslogtreecommitdiff
path: root/exercise03
diff options
context:
space:
mode:
Diffstat (limited to 'exercise03')
-rw-r--r--exercise03/exercise03.tex521
1 files changed, 521 insertions, 0 deletions
diff --git a/exercise03/exercise03.tex b/exercise03/exercise03.tex
new file mode 100644
index 0000000..be192ce
--- /dev/null
+++ b/exercise03/exercise03.tex
@@ -0,0 +1,521 @@
+% SPDX-License-Identifier: CC-BY-SA-4.0
+%
+% Copyright (c) 2020 Philipp Le
+%
+% Except where otherwise noted, this work is licensed under a
+% Creative Commons Attribution-ShareAlike 4.0 License.
+%
+% Please find the full copy of the licence at:
+% https://creativecommons.org/licenses/by-sa/4.0/legalcode
+
+\phantomsection
+\addcontentsline{toc}{section}{Exercise 3}
+\section*{Exercise 3}
+
+%%%%%%%%%%%%%%%%%%%%%%%%%%%%%%%%%%%%%%%%%%%%%%%%%%%%%%%%%%%%%%%%%%%%%%%%%%%%%%%
+\begin{question}[subtitle={Stochastic Process}]
+ A normally distributed random process produces the sequences $x_1(t)$, $x_2(t)$ and $x_3(t)$.
+ \begin{table}[H]
+ \centering
+ \begin{tabular}{|l|r|r|r|r|r|r|r|r|r|r|}
+ \hline
+ $t$ & 0 & 1 & 2 & 3 & 4 & 5 & 6 & 7 & 8 & 9 \\
+ \hline
+ \hline
+ $x_1(t)$ & 4.99 & 4.37 & 8.57 & 4.01 & 3.77 & 3.35 & 3.87 & 8.39 & 6.89 & 1.96 \\
+ \hline
+ $x_2(t)$ & 3.95 & 5.35 & 2.94 & 6.38 & 4.78 & 7.62 & 5.25 & 6.81 & 5.65 & 5.29 \\
+ \hline
+ $x_3(t)$ & 7.01 & 4.40 & 4.26 & 6.54 & 4.53 & 6.85 & 4.46 & 5.81 & 6.49 & 4.11 \\
+ \hline
+ \end{tabular}
+ \end{table}
+ \begin{tasks}
+ \task
+ Calculate the stochastic mean for each time instance!
+
+ \task
+ Calculate the temporal mean for each sequence!
+
+ \task
+ The process is ergodic with $\mu_x = 5.00$. However, why is the condition $\E\left\{\vect{x}\right\} = \overline{x} = \mu_x$ not fulfilled?
+ \end{tasks}
+\end{question}
+
+\begin{solution}
+ \begin{tasks}
+ \task
+ \begin{table}[H]
+ \centering
+ \begin{tabular}{|l|r|r|r|r|r|r|r|r|r|r|}
+ \hline
+ $t$ & 0 & 1 & 2 & 3 & 4 & 5 & 6 & 7 & 8 & 9 \\
+ \hline
+ \hline
+ $\E\left\{\vect{x}(t)\right\}$ & 5.32 & 4.71 & 5.26 & 5.64 & 4.36 & 5.94 & 4.53 & 7.00 & 6.34 & 3.79 \\
+ \hline
+ \end{tabular}
+ \end{table}
+
+ \task
+ \begin{itemize}
+ \item $\overline{x_1} = 5.01$
+ \item $\overline{x_1} = 5.40$
+ \item $\overline{x_1} = 5.45$
+ \end{itemize}
+
+ \task
+ \begin{itemize}
+ \item There are only $N = 3$ sequences drawn from the random process. $\E\left\{\vect{x}\right\}$ will converge to $5.00$ for $N \rightarrow \infty$. $N = 3$ is too short.
+ \item Each sequence is only $L = 10$ samples long. $\overline{x}$ will converge to $5.00$ for $L \rightarrow \infty$. $L = 10$ is too short.
+ \end{itemize}
+ \end{tasks}
+\end{solution}
+
+%%%%%%%%%%%%%%%%%%%%%%%%%%%%%%%%%%%%%%%%%%%%%%%%%%%%%%%%%%%%%%%%%%%%%%%%%%%%%%%
+\begin{question}[subtitle={Cross-Correlation and Autocorrelation}]
+ Two signals are given $f_1(t)$ and $f_2(t)$. The signals are value- and time-continuous. Ten samples are given. Both signals are zero for $t < -5$ and $t > 5$.
+ \begin{table}[H]
+ \centering
+ \begin{tabular}{|l|r|r|r|r|r|r|r|r|r|r|r|}
+ \hline
+ $t$ & -5 & -4 & -3 & -2 & -1 & 0 & 1 & 2 & 3 & 4 & 5 \\
+ \hline
+ \hline
+ $f_1(t)$ & 0 & 1 & 2 & 3 & 4 & 5 & 4 & 3 & 2 & 1 & 0 \\
+ $f_2(t)$ & -1.34 & 0.30 & 14.54 & -1.54 & -3.03 & -1.72 & 14.16 & 2.70 & 1.17 & -2.44 & -4.66 \\
+ \hline
+ \end{tabular}
+ \end{table}
+ \begin{tasks}
+ \task
+ Calculate the cross-correlation
+ \begin{equation*}
+ \mathrm{R}_{f_1 f_2}(\tau) \approx \left(f_1 \star f_2\right)(\tau) = \sum\limits_{t=-5}^{5} f_1(t) f_2(t + \tau)
+ \end{equation*}
+ for $t = -2$, $t = -1$ and $t = 0$.
+
+ \task
+ Signal $f_2(t)$ contains signal $f_1(t)$ which is superimposed by noise. Using the values calculated in a), how much is the time $\Delta t$ lag between $f_1(t)$ and $f_2(t)$?
+
+ \task
+ Calculate the autocorrelation
+ \begin{equation*}
+ \mathrm{R}_{f_1 f_1}(\tau) \approx \left(f_1 \star f_1\right)(\tau) = \sum\limits_{t=-5}^{5} f_1(t) f_1(t + \tau)
+ \end{equation*}
+ for $t = -2$, $t = -1$, $t = 0$, $t = 1$ and $t = 2$.
+
+ \task
+ How much is the signal energy $E$ of $f_1(t)$?
+ \begin{equation*}
+ E \approx \frac{1}{T} \sum\limits_{t=-5}^{5} \left|f_1(t)\right|^2
+ \end{equation*}
+ with $T = 11$?
+ \end{tasks}
+
+ \textit{Remark:} Only samples of the functions with a spacing of $1$ are given. Therefore, the indefinite integrals of both cross-correlation and autocorrelation can be approximated using the sums.
+\end{question}
+
+\begin{solution}
+ \begin{tasks}
+ \task
+ Full cross-correlation between $-10 \geq t \geq 10$. The cross-correlation is zero everywhere outside of the interval.
+ \begin{table}[H]
+ \centering
+ \begin{tabular}{|l|r|r|r|r|r|r|r|r|r|}
+ \hline
+ $\tau$ & -10 & -9 & -8 & -7 & -6 & -5 & -4 & -3 & -2 \\
+ \hline
+ $\mathrm{R}_{f_1 f_2}(\tau)$ & 0.0 & -4.66 & -11.76 & -17.69 & -20.92 & -9.99 & 8.54 & 28.92 & 45.42 \\
+ \hline
+ \hline
+ $\tau$ & -1 & 0 & 1 & 2 & 3 & 4 & 5 & 6 & 7 \\
+ \hline
+ $\mathrm{R}_{f_1 f_2}(\tau)$ & 71.06 & 68.68 & 63.74 & 62.42 & 65.35 & 41.9 & 32.01 & 23.08 & 11.12 \\
+ \hline
+ \hline
+ $\tau$ & 8 & 9 & 10 & & & & & & \\
+ \hline
+ $\mathrm{R}_{f_1 f_2}(\tau)$ & -2.38 & -1.34 & 0.0 & & & & & & \\
+ \hline
+ \end{tabular}
+ \end{table}
+ \begin{itemize}
+ \item $\left(f_1 \star f_2\right)(-2) = 45.42$
+ \item $\left(f_1 \star f_2\right)(-1) = 71.06$
+ \item $\left(f_1 \star f_2\right)(0) = 68.68$
+ \end{itemize}
+
+ \task
+ \begin{itemize}
+ \item The maximum value is at $\tau = -1$.
+ \item $f_2(t)$ is advanced by $\Delta t = 1$ in relation to $f_1(t)$.
+ \end{itemize}
+
+ \task
+ Full autocorrelation between $-10 \geq t \geq 10$. The autocorrelation is zero everywhere outside of the interval.
+ \begin{table}[H]
+ \centering
+ \begin{tabular}{|l|r|r|r|r|r|r|r|r|r|r|r|}
+ \hline
+ $\tau$ & -10 & -9 & -8 & -7 & -6 & -5 & -4 & -3 & -2 & -1 & 0 \\
+ \hline
+ $\mathrm{R}_{f_1 f_1}(\tau)$ & 0 & 0 & 1 & 4 & 10 & 20 & 3. & 52 & 68 & 80 & 85 \\
+ \hline
+ \hline
+ $\tau$ & 1 & 2 & 3 & 4 & 5 & 6 & 7 & 8 & 9 & 10 & \\
+ \hline
+ $\mathrm{R}_{f_1 f_1}(\tau)$ & 80 & 68 & 52 & 35 & 20 & 10 & 4 & 1 & 0 & 0 & \\
+ \hline
+ \end{tabular}
+ \end{table}
+ Only three values must be calculated:
+ \begin{itemize}
+ \item $\mathrm{R}_{f_1 f_1}(\tau)(-2) = 68$
+ \item $\mathrm{R}_{f_1 f_1}(\tau)(-1) = 80$
+ \item $\mathrm{R}_{f_1 f_1}(\tau)(0) = 85$
+ \end{itemize}
+ The other two values can be deducted from the symmetry rules:
+ \begin{itemize}
+ \item $\mathrm{R}_{f_1 f_1}(\tau)(1) = \mathrm{R}_{f_1 f_1}(\tau)(-1) = 80$
+ \item $\mathrm{R}_{f_1 f_1}(\tau)(2) = \mathrm{R}_{f_1 f_1}(\tau)(-2) = 68$
+ \end{itemize}
+
+ \task
+ \begin{itemize}
+ \item It is not necessary to solve the sum again.
+ \item The sum is $\mathrm{R}_{f_1 f_1}(\tau)(0)$.
+ \item The energy is
+ \begin{equation*}
+ E = \frac{1}{11} \mathrm{R}_{f_1 f_1}(\tau)(0) = 7.23
+ \end{equation*}
+ \end{itemize}
+ \end{tasks}
+\end{solution}
+
+
+%%%%%%%%%%%%%%%%%%%%%%%%%%%%%%%%%%%%%%%%%%%%%%%%%%%%%%%%%%%%%%%%%%%%%%%%%%%%%%%
+\begin{question}[subtitle={Power Spectral Density}]
+ Are the following statements about the power spectral density (PSD) true or false? If false, correct the statement!
+ \begin{tasks}
+ \task
+ The PSD is measure for the fraction of power which is located at a certain frequency.
+
+ \task
+ The unit of the PSD is the same as the signal.
+
+ \task
+ The PSD is even, if the signal is purely real-valued.
+
+ \task
+ The PSD is odd, if the signal is complex-valued.
+
+ \task
+ The PSD is always real-valued.
+
+ \task
+ The PSD can be used to determine the output signal of an LTI system in amplitude and phase.
+
+ \end{tasks}
+\end{question}
+
+\begin{solution}
+ \begin{tasks}
+ \task
+ True. The signal power can be obtained by integration of the PSD over all frequencies.
+ \begin{equation*}
+ P = \int\limits_{-\infty}^{\infty} \mathrm{S}_{xx}(\omega) \; \mathrm{d} \omega
+ \end{equation*}
+
+ \task
+ False.
+ \begin{equation*}
+ \left(\text{Unit of PSD}\right) = \frac{\left(\text{Unit of signal}\right)^2}{\left(\text{Unit of frequency}\right)}
+ \end{equation*}
+
+ For example \si{W/Hz}. Never \si{W} only, because it is the density of the power across the frequency.
+
+ \task
+ True
+
+ \task
+ False. There are no symmetry rules for the PSD, if the signal is complex-valued.
+
+ \task
+ True.
+ \begin{itemize}
+ \item The autocorrelation function is always Hermitian.
+ \item The Fourier transform of a Hermitian function is always real-valued.
+ \item Therefore, the PSD is real-valued.
+ \end{itemize}
+
+ \task
+ False.
+ \begin{itemize}
+ \item Only the amplitude can be calculated.
+ \item The phase information of the system is lost by squaring the absolute value of the transfer funtion.
+ \item Furthermore, the PSD does not contain phase information of the signal.
+ \end{itemize}
+ \begin{equation*}
+ \mathrm{S}_{yy}(\omega) = \left|\underline{H}\left(j \omega\right)\right|^2 \cdot \mathrm{S}_{xx}(\omega)
+ \end{equation*}
+ \end{tasks}
+\end{solution}
+
+
+%%%%%%%%%%%%%%%%%%%%%%%%%%%%%%%%%%%%%%%%%%%%%%%%%%%%%%%%%%%%%%%%%%%%%%%%%%%%%%%
+\begin{question}[subtitle={Decibel}]
+ \begin{tasks}
+ \task
+ Convert to logarithmic scale or vice versa!
+ \begin{enumerate}
+ \item \SI{2}{mW}, $P_0 = \SI{1}{mW}$
+ \item \SI{2}{V}, $U_0 = \SI{1}{V}$
+ \item \num{400000}
+ \item \SI{5}{GHz}, $f_0 = \SI{1}{Hz}$
+ \item \SI{8}{fW/MHz}, $P_0 = \SI{1}{mW}$
+ \item \SI{5}{dB\micro\volt}
+ \item \SI{-10}{dBm}
+ \end{enumerate}
+
+ \task
+ Convert to logarithmic scale or vice versa!
+ \begin{enumerate}
+ \item $1$
+ \item $2$
+ \item $4$
+ \item $0.5$
+ \item $0.25$
+ \item $10$
+ \item $100$
+ \item $0.1$
+ \item $0.01$
+ \item $500$
+ \item $0.04$
+ \end{enumerate}
+
+ \task
+ A signal is transmitted with a power of \SI{13}{dBm}. The signal is attenuated by \SI{86}{dB} on its way to the receiver. Assume that a ohmic resistance of \SI{50}{\ohm} is connected to the antenna. What voltage can be measured at this resistance? Give your result in linear and logarithmic scale!
+ \end{tasks}
+\end{question}
+
+\begin{solution}
+ \begin{tasks}
+ \task
+ \begin{enumerate}
+ \item $\SI{10}{dBm} \cdot \log_{10} \left(\frac{\SI{2}{mW}}{\SI{1}{mW}}\right) = \SI{3}{dBm}$
+ \item $\SI{20}{dBV} \cdot \log_{10} \left(\frac{\SI{2}{V}}{\SI{1}{V}}\right) = \SI{6}{dBV}$ (20 instead of 10 for voltages and currents)
+ \item $\SI{10}{dB} \cdot \log_{10} \left(400000\right) = \SI{53}{dB}$
+ \item $\SI{10}{dBHz} \cdot \log_{10} \left(\frac{\SI{5}{GHz}}{\SI{1}{Hz}}\right) = \SI{97}{dbHz}$
+ \item $\SI{10}{dBm/Hz} \cdot \log_{10} \left(\frac{\SI{8}{fW}}{\SI{1}{mW}}\right) = \SI{-111}{dB/Hz}$
+ \item $\SI{1}{\micro\volt} \cdot 10^{\SI{5}{dB\micro\volt} / 20} = \SI{1.8}{\micro\volt}$
+ \item $\SI{1}{mW} \cdot 10^{\SI{-10}{dBm} / 10} = \SI{100}{\micro.W}$
+ \end{enumerate}
+
+ \task
+ \begin{enumerate}
+ \item $\SI{0}{dB}$
+ \item $\SI{3}{dB}$
+ \item $\SI{6}{dB}$, Hint: $4 = 2 \cdot 2 \equiv \SI{3}{dB} + \SI{3}{dB} = \SI{6}{dB}$
+ \item $\SI{-3}{dB}$
+ \item $\SI{-6}{dB}$, Hint: $0.25 = 0.5 \cdot 0.5 \equiv \SI{-3}{dB} + \SI{-3}{dB} = \SI{-6}{dB}$
+ \item $\SI{10}{dB}$
+ \item $\SI{20}{dB}$
+ \item $\SI{-10}{dB}$
+ \item $\SI{-20}{dB}$
+ \item $\SI{27}{dB}$, Hint: $50 = 10 \cdot 10 \cdot 10 \cdot 0.5 \equiv \SI{10}{dB} + \SI{10}{dB} + \SI{10}{dB} - \SI{3}{dB} = \SI{27}{dB}$
+ \item $\SI{-14}{dB}$, Hint: $0.04 = 0.1 \cdot 0.1 \cdot 2 \cdot 2 \equiv \SI{-10}{dB} + \SI{-10}{dB} + \SI{3}{dB} + \SI{3}{dB} = \SI{-14}{dB}$
+ \end{enumerate}
+ Remember these rules as an RF engineer! You will need them often. ;)
+
+ \task
+ Power at the receiver antenna:
+ \begin{equation*}
+ \SI{13}{dBm} \underbrace{- \SI{86}{dB}}_{\text{Atennuation}} = \SI{-73}{dBm}
+ \end{equation*}
+
+ \begin{itemize}
+ \item 1st way: Convert to linear scale, then back to logarithmic scale
+ \begin{equation*}
+ \begin{split}
+ \SI{-73}{dBm} &\equiv \SI{50.1}{pW} \\
+ U = \sqrt{P \cdot R} &= \sqrt{\SI{50.1}{pW} \cdot \SI{50}{\ohm}} = \SI{50}{\micro\volt} \\
+ \SI{50}{\micro\volt} &\equiv \SI{17}{dB\micro\volt}
+ \end{split}
+ \end{equation*}
+ \item 2nd way: Convert everything to logarithmic scale. The multiplication of power and resistance must happen with ``pure'' SI units.
+ \begin{equation*}
+ \begin{split}
+ \SI{50}{\ohm} &\equiv \SI{10}{dB\ohm} \cdot \log_{10} \left(\frac{\SI{50}{\ohm}}{\SI{1}{\ohm}}\right) = \SI{17}{dB\ohm} \\
+ \SI{-73}{dBm} \underbrace{- \SI{30}{dB}}_{\text{\si{mW} to \si{W}}} &= \SI{-103}{dBW} \quad \text{SI unit!} \\
+ \SI{-103}{dBW} + \SI{17}{dB\ohm} &= \SI{-86}{dBV} \quad \text{SI unit!} \\
+ \SI{-86}{dBV} \underbrace{+ \SI{120}{dB}}_{\text{\si{V} to \si{\micro\volt}}} &= \SI{34}{dB\micro\volt} \\
+ \SI{34}{dB\micro\volt} &\equiv \SI{50}{\micro\volt}
+ \end{split}
+ \end{equation*}
+ \end{itemize}
+ \end{tasks}
+\end{solution}
+
+%%%%%%%%%%%%%%%%%%%%%%%%%%%%%%%%%%%%%%%%%%%%%%%%%%%%%%%%%%%%%%%%%%%%%%%%%%%%%%%
+\begin{question}[subtitle={Noise}]
+ The following system is given:
+ \begin{figure}[H]
+ \centering
+ \begin{adjustbox}{scale=0.7}
+ \begin{tikzpicture}
+ \node[draw, block] (BPF){Band-pass filter\\ $L_{G,1} = \SI{-3}{dB}$\\ $L_{F,1} = \SI{3}{dB}$};
+ \node[draw, block, right=of BPF] (Mix){Mixer\\ $L_{G,2} = \SI{3}{dB}$\\ $L_{F,2} = \SI{20}{dB}$};
+ \node[draw, block, right=of Mix] (Demod){Demodulator\\ $L_{G,3} = \SI{6}{dB}$\\ $L_{F,3} = \SI{10}{dB}$};
+ \node[draw, block, right=of Demod] (Amp){Amplifier\\ $L_{G,4} = \SI{50}{dB}$\\ $L_{F,4} = \SI{16}{dB}$};
+
+ \draw[o->] ([xshift=-1cm]BPF.west) node[left,align=right]{Input $u(t)$} -- (BPF.west);
+ \draw[->] (BPF.east) -- (Mix.west);
+ \draw[->] (Mix.east) -- (Demod.west);
+ \draw[->] (Demod.east) -- (Amp.west);
+ \draw[->] (Amp.east) -- ([xshift=1cm]Amp.east) node[right,align=left]{Output};
+ \end{tikzpicture}
+ \end{adjustbox}
+ \end{figure}
+ \begin{itemize}
+ \item The band-pass filter has an input impedance of $\SI{50}{\ohm}$.
+ \item The band-pass filter has a bandwidth of $\SI{2}{MHz}$.
+ \item All system components are at room temperature: \SI{25}{\degreeCelsius}.
+ \end{itemize}
+ The input signal is:
+ \begin{equation*}
+ u(t) = \SI{70.71}{\micro\volt} \cdot \cos\left(\omega_0 t\right)
+ \end{equation*}
+
+ Give all results in the logarithmic scale!
+ \begin{tasks}
+ \task
+ What is the signal power, the thermal noise power and the SNR at the input?
+
+ \task
+ What is the overall noise figure and overall gain of the chain? What is the signal power, the thermal noise power and the SNR at the output of the chain?
+
+ \task
+ Now a low noise amplifier is placed in front of the mixer. The gain of the last amplifier is reduced, so that the overall gain does not change.
+ \begin{figure}[H]
+ \centering
+ \begin{adjustbox}{scale=0.5}
+ \begin{tikzpicture}
+ \node[draw, block] (Amp){Low noise amplifier\\ $L_{G,0} = \SI{30}{dB}$\\ $L_{F,0} = \SI{6}{dB}$};
+ \node[draw, block, right=of Amp] (BPF){Band-pass filter\\ $L_{G,1} = \SI{-3}{dB}$\\ $L_{F,1} = \SI{3}{dB}$};
+ \node[draw, block, right=of BPF] (Mix){Mixer\\ $L_{G,2} = \SI{3}{dB}$\\ $L_{F,2} = \SI{20}{dB}$};
+ \node[draw, block, right=of Mix] (Demod){Demodulator\\ $L_{G,3} = \SI{6}{dB}$\\ $L_{F,3} = \SI{10}{dB}$};
+ \node[draw, block, right=of Demod] (Amp2){Amplifier\\ $L_{G,4} = \SI{20}{dB}$\\ $L_{F,4} = \SI{16}{dB}$};
+
+ \draw[o->] ([xshift=-1cm]Amp.west) node[left,align=right]{Input $u(t)$} -- (Amp.west);
+ \draw[->] (Amp.east) -- (BPF.west);
+ \draw[->] (BPF.east) -- (Mix.west);
+ \draw[->] (Mix.east) -- (Demod.west);
+ \draw[->] (Demod.east) -- (Amp2.west);
+ \draw[->] (Amp2.east) -- ([xshift=1cm]Amp2.east) node[right,align=left]{Output};
+ \end{tikzpicture}
+ \end{adjustbox}
+ \end{figure}
+ For this new constellation: What is the overall noise figure and overall gain of the chain? What is the signal power, the thermal noise power and the SNR at the output of the chain?
+ \end{tasks}
+\end{question}
+
+\begin{solution}
+ \begin{tasks}
+ \task
+ \begin{itemize}
+ \item The RMS value of the signals is: $\frac{\SI{70.71}{\micro\volt}}{\sqrt{2}} = \SI{50}{\micro\volt}$
+ \item At $\SI{50}{\ohm}$, this yields a power of: $P = \frac{U^2}{R} = \SI{50}{pW} \equiv L_{P,S} = \SI{-73}{dBm}$
+ \item The noise floor at room temperature is: $k_B T \equiv \SI{-174}{dBm/Hz}$
+ \item The filter's bandwidth is: $\SI{63}{dBHz}$
+ \item The noise power is: $k_B T \Delta f \equiv L_{P,N} = \SI{-174}{dBm/Hz} + \SI{63}{dBHz} = \SI{-111}{dBm}$
+ \item The SNR is: $L_{\mathrm{SNR}} = L_{P,S} - L_{P,N} = \SI{-73}{dBm} - (\SI{-111}{dBm}) = \SI{38}{dBm}$
+ \end{itemize}
+
+ \task
+ Convert to linear scale:
+ \begin{itemize}
+ \item $G_1 = 0.5$
+ \item $F_1 = 2$
+ \item $G_2 = 2$
+ \item $F_2 = 100$
+ \item $G_3 = 4$
+ \item $F_3 = 10$
+ \item $G_4 = 100000$
+ \item $F_4 = 40$
+ \end{itemize}
+
+ Using the Friis formula:
+ \begin{equation*}
+ \begin{split}
+ F_{\text{total}} &= F_1 + \frac{F_2 - 1}{G_1} + \frac{F_3 - 1}{G_1 G_2} + \frac{F_4 - 1}{G_1 G_2 G_3} \\
+ &= 218 \\
+ L_{F,\text{total}} &= \SI{23.4}{dB}
+ \end{split}
+ \end{equation*}
+ Overall gain:
+ \begin{equation*}
+ L_{G,\text{total}} = \SI{-3}{dB} + \SI{3}{dB} + \SI{6}{dB} + \SI{50}{dB} = \SI{56}{dB}
+ \end{equation*}
+ \begin{itemize}
+ \item Output signal power: $\SI{-73}{dBm} + L_{G,\text{total}} = \SI{-17}{dBm}$
+ \item Output noise power: $\SI{-111}{dBm} + L_{G,\text{total}} + L_{F,\text{total}} = \SI{-31.6}{dBm}$
+ \item Output SNR: $\SI{38}{dBm} - L_{F,\text{total}} = \SI{14.6}{dB}$
+ \end{itemize}
+
+ \task
+ Convert to linear scale:
+ \begin{itemize}
+ \item $G_0 = 1000$
+ \item $F_0 = 4$
+ \item $G_1 = 0.5$
+ \item $F_1 = 2$
+ \item $G_2 = 2$
+ \item $F_2 = 100$
+ \item $G_3 = 4$
+ \item $F_3 = 10$
+ \item $G_4 = 10$
+ \item $F_4 = 40$
+ \end{itemize}
+
+ Using the Friis formula:
+ \begin{equation*}
+ \begin{split}
+ F_{\text{total}} &= F_0 + \frac{F_1 - 1}{G_0} + \frac{F_2 - 1}{G_0 G_1} + \frac{F_3 - 1}{G_0 G_1 G_2} + \frac{F_4 - 1}{G_0 G_1 G_2 G_3} \\
+ &= 4.22 \\
+ L_{F,\text{total}} &= \SI{6.3}{dB}
+ \end{split}
+ \end{equation*}
+ Overall gain:
+ \begin{equation*}
+ L_{G,\text{total}} = \SI{50}{dB} + \SI{-3}{dB} + \SI{3}{dB} + \SI{6}{dB} = \SI{56}{dB}
+ \end{equation*}
+ \begin{itemize}
+ \item Output signal power: $\SI{-73}{dBm} + L_{G,\text{total}} = \SI{-17}{dBm}$
+ \item Output noise power: $\SI{-111}{dBm} + L_{G,\text{total}} + L_{F,\text{total}} = \SI{-48.7}{dBm}$
+ \item Output SNR: $\SI{38}{dBm} - L_{F,\text{total}} = \SI{31.7}{dB}$
+ \end{itemize}
+
+ \textbf{Conclusion:}
+ \begin{itemize}
+ \item Placing the low noise amplifier in front of the chain, gives a much better ($\SI{16.1}{dB}$) SNR. The gain is equal.
+ \item The overall noise figure is dominated by the first component in the chain.
+ \item In this constellation, first component is a low noise amplifier with high gain and low noise figure.
+ \item The high gain scales down the noise contribution of the components following in the chain.
+ \item Therefore, it is always feasible to place a low noise amplifier (high gain, low noise) to the front of the chain.
+ \end{itemize}
+ \end{tasks}
+\end{solution}
+
+
+
+%%%%%%%%%%%%%%%%%%%%%%%%%%%%%%%%%%%%%%%%%%%%%%%%%%%%%%%%%%%%%%%%%%%%%%%%%%%%%%%
+%\begin{question}[subtitle={Decibel}]
+% \begin{tasks}
+% \end{tasks}
+%\end{question}
+%
+%\begin{solution}
+% \begin{tasks}
+% \end{tasks}
+%\end{solution}